Solving Multivariable Limits: Tips & Tricks

Click For Summary
The discussion focuses on solving two multivariable limits. For the first limit, participants clarify that as y approaches 0, e^y approaches 1, which simplifies the evaluation. The second limit is debated, with insights on how approaching along different paths can indicate whether the limit exists; specifically, if a path leads to undefined behavior, the limit does not exist. The importance of testing multiple paths to confirm limit existence is emphasized, and it is concluded that substituting y=x^3 is not valid for determining the limit. Overall, the conversation highlights key strategies for tackling multivariable limits effectively.
Wesleytf
Messages
31
Reaction score
0
I'm taking multi-variable after having a while off from school, so forgive me if these are simple ones that I just don't "see"

Homework Statement


lim (x, y) --> 0, 0 \frac{x^2 y^2 e^y}{x^4+4y^2}

and

lim (x, y) --> (1, 1) \frac{x-y}{x^3-y}

Homework Equations


The Attempt at a Solution



The bottom one I feel doesn't exist because as x->0+, the ^3 is making it larger, where as when x->0-, the ^3 is making it smaller. I know this is poor logic; it's just a gut feeling about it. substituting y=mx or similar didn't get me anywhere. Plugging obviously doesn't work. I don't see anyway to simplify, but maybe there is a way. I also don't think polar coordinates will work for either.

I really think I only need a hint to the method of solution, so don't go solving the whole thing for me.
 
Last edited:
Physics news on Phys.org
For the bottom one, suppose you approach along the path y=x^3? For the top one you don't have to worry about the factor e^y, why not? Can you handle the rest?
 
Dick said:
For the bottom one, suppose you approach along the path y=x^3? For the top one you don't have to worry about the factor e^y, why not? Can you handle the rest?

For the top one, e^y -->1 as y->0. I wasn't sure if saying that and then doing the rest of the limit was a legal move. Should be easy now (I see one way by polar coordinates and then squeeze theorem, I think)

For the bottom one, I tried that, but doesn't that just 'break' the function--as in, we can't tell what it will be by that method? The way I thought the "two different limits for two different paths" property worked was that it would only work if you found two actual different paths. Taking y=x^3 makes one path not exist; is that enough to make the limit of the function not exist?
 
Wesleytf said:
For the top one, e^y -->1 as y->0. I wasn't sure if saying that and then doing the rest of the limit was a legal move. Should be easy now (I see one way by polar coordinates and then squeeze theorem, I think)

For the bottom one, I tried that, but doesn't that just 'break' the function--as in, we can't tell what it will be by that method? The way I thought the "two different limits for two different paths" property worked was that it would only work if you found two actual different paths. Taking y=x^3 makes one path not exist; is that enough to make the limit of the function not exist?

If the function has a limit, it has to approach that limit along all paths. If the function doesn't even exist along a path, then the limit doesn't exist. If you think about what's happening 'close' to the path y=x^3 then you'll see the function becomes large without bound.
 
Dick said:
If the function has a limit, it has to approach that limit along all paths. If the function doesn't even exist along a path, then the limit doesn't exist. If you think about what's happening 'close' to the path y=x^3 then you'll see the function becomes large without bound.
ha, I had already went back and put it on paper, and as soon as I did the behavior 'close' to y=x^3 became clear. thanks! hopefully my brain will uncrustify itself soon...
 
follow up on the bottom one: Subbing y=x^3 will not work because it is not within the definition of a limit. However, comparing the substitutions y=x and y=1 does work to prove that it indeed DNE.
 
Question: A clock's minute hand has length 4 and its hour hand has length 3. What is the distance between the tips at the moment when it is increasing most rapidly?(Putnam Exam Question) Answer: Making assumption that both the hands moves at constant angular velocities, the answer is ## \sqrt{7} .## But don't you think this assumption is somewhat doubtful and wrong?

Similar threads

Replies
5
Views
2K
  • · Replies 5 ·
Replies
5
Views
2K
Replies
7
Views
2K
  • · Replies 2 ·
Replies
2
Views
1K
  • · Replies 10 ·
Replies
10
Views
2K
  • · Replies 8 ·
Replies
8
Views
2K
Replies
9
Views
2K
Replies
7
Views
1K
  • · Replies 8 ·
Replies
8
Views
2K
  • · Replies 8 ·
Replies
8
Views
2K